Difference between revisions of "1952 AHSME Problems/Problem 43"

(Problem)
(Problem)
Line 4: Line 4:
 
<math>\textbf{(A) } \qquad</math> equal to the semi-circumference of the original circle
 
<math>\textbf{(A) } \qquad</math> equal to the semi-circumference of the original circle
 
<math>\textbf{(B) } \qquad</math> equal to the diameter of the original circle
 
<math>\textbf{(B) } \qquad</math> equal to the diameter of the original circle
<math>\textbf{(C) } (11 - k) \qquad</math> greater than the diameter, but less than the semi-circumference of the original circle
+
<math>\textbf{(C) } \qquad</math> greater than the diameter, but less than the semi-circumference of the original circle
<math>\textbf{(D) } (k - 1) \qquad</math> that is infinite
+
<math>\textbf{(D) } \qquad</math> that is infinite
<math>\textbf{(E) } (k+1) </math> greater than the semi-circumference
+
<math>\textbf{(E) } </math> greater than the semi-circumference
  
 
== Solution ==
 
== Solution ==

Revision as of 08:40, 1 May 2016

Problem

The diameter of a circle is divided into $n$ equal parts. On each part a semicircle is constructed. As $n$ becomes very large, the sum of the lengths of the arcs of the semicircles approaches a length: $\textbf{(A) } \qquad$ equal to the semi-circumference of the original circle $\textbf{(B) } \qquad$ equal to the diameter of the original circle $\textbf{(C) } \qquad$ greater than the diameter, but less than the semi-circumference of the original circle $\textbf{(D) }  \qquad$ that is infinite $\textbf{(E) }$ greater than the semi-circumference

Solution

Let our two digit number be $AB$. Its value is $10A + B$. The number formed by interchanging its digits is BA and has value $10B + A$. Setting AB equal to $k$ times the sum of the digits yields \[10A + B = k(A + B)\] We now must relate AB to BA. Note that \[11(A + B) - (10A + B) = 10B + A\] Using this in the first equation yields \[10A + B = k(A + B)\] \[11(A + B) - (10A + B) = 11(A + B) - k(A + B)\] \[10B + A = (11 - k)(A + B)\] Therefore, $BA$ is $(11 - k)$ times the sum of its digits and our answer is $\fbox{C}$.

See also

1952 AHSC (ProblemsAnswer KeyResources)
Preceded by
Problem 42
Followed by
Problem 44
1 2 3 4 5 6 7 8 9 10 11 12 13 14 15 16 17 18 19 20 21 22 23 24 25 26 27 28 29 30 31 32 33 34 35 36 37 38 39 40 41 42 43 44 45 46 47 48 49 50
All AHSME Problems and Solutions

The problems on this page are copyrighted by the Mathematical Association of America's American Mathematics Competitions. AMC logo.png